Sports - Lower Extremity (Complete) Flashcards

1
Q

What are the arthroscopic hip portals and structures at risk?

[AAOS comprehensive review 2, 2014]

A
  1. Anterior
  • Lateral femoral cutaneous nerve
  • Femoral nerve
  • Femoral artery
  1. Anterolateral
    * Superior gluteal nerve
  2. Posterolateral
    * Sciatic nerve
  3. Midanterior
    * Lateral femoral cutaneous nerves
How well did you know this?
1
Not at all
2
3
4
5
Perfectly
2
Q

During hip arthroscopy, what nerve is at risk due to traction and the perineal post?

[AAOS comprehensive review 2, 2014]

A

Pudendal nerve

  • Can result in:
    • Hypoaesthesia of the perineum, scrotum and glans penis
    • Erectile dysfunction
    • Urinary incontinence
How well did you know this?
1
Not at all
2
3
4
5
Perfectly
3
Q

What are the indications for hip arthroscopy?

[Sports Health. 2017; 9(5): 402–413.]

A
  1. Central compartment
  • Labral tears
  • Chondral pathology
  • Ligamentum teres pathology
  • Septic arthritis
  • Loose bodies
    2. Peripheral compartment
  • Femoroacetabular impingement
  • Subspine impingement
  • Synovial disorders
  • Capsular disorders
  • Psoas tendon disorders
    3. Peritrochanteric compartment
  • Greater trochanteric pain syndrome
  • External snapping hip/iliotibial band disorder
  1. Deep gluteal space
  • Ischiofemoral impingement
  • Proximal hamstring disorders
  • Sciatic nerve disorders
How well did you know this?
1
Not at all
2
3
4
5
Perfectly
4
Q

What are the contraindications to hip arthroscopy?

[Sports Health. 2017; 9(5): 402–413.]

A
  1. Advanced OA
  2. Ankylosis
  3. Acetabular and/or femoral dysplasia
  4. Severe deformity
    * Retroversion, SCFE, Perthes
  5. Obesity (relative)
  6. Neurological injuries/disorders (relative)
    * Eg. pudendal neuralgia or peroneal or sciatic nerve palsy
How well did you know this?
1
Not at all
2
3
4
5
Perfectly
5
Q

What are the most common complications following hip arthroscopy?

[Bone Joint J. 2017 Dec;99-B(12):1577-1583]

A
  1. Nerve injury (0.9%)
  • Pudendal > LFCN > sciatic > common peroneal > femoral
  • Traction injuries include sciatic, common peroneal and femoral [Muscles Ligaments Tendons J. 2016 Jul-Sep; 6(3): 402–409.]
  • Compression injuries include pudendal nerve
  • Portal placement injuries include LFCN
    2. Iatrogenic injury (0.7%)
  • Chondral > labral
    3. HO (0.6%)
    4. Adhesions (0.2%)
    5. Infection (0.2%)
  • Superficial > deep
    6. Other
  • DVT, perineal skin damage, hematoma, broken instrument, incomplete reshaping, femoral neck fracture, hip instability, iliopsoas tendinitis, AVN, ankle pain, arthrofibrosis, dislocation
How well did you know this?
1
Not at all
2
3
4
5
Perfectly
6
Q

What is the most common major complication following hip arthroscopy?

[Bone Joint J. 2017 Dec;99-B(12):1577-1583]

A

Intra-abdominal fluid extravasation

How well did you know this?
1
Not at all
2
3
4
5
Perfectly
7
Q

What is the innervation of the acetabular labrum?

[BMC Musculoskeletal Disorders 2014, 15:41]

A

Branch from nerve to quadratus femoris and obturator nerve

  • Contains:
    • Free nerve endings for nociception
    • Nerve end organs (Pacini, Golgi, Ruffini corpuscles) for proprioception
  • Higher concentration in:
    • Anterosuperior and postersuperior labrum
    • Articular side more so than the capsular side
How well did you know this?
1
Not at all
2
3
4
5
Perfectly
8
Q

What is the blood supply to the acetabular labrum?

[J Bone Joint Surg Am. 2010 Nov 3;92(15):2570-5]

A

Periacetabular vascular ring

  • Originates from
    • Superior and inferior gluteal vessels
    • Medial and lateral femoral circumflex arteries
    • Intrapelvic vascular system.
How well did you know this?
1
Not at all
2
3
4
5
Perfectly
9
Q

What is the function of the hip labrum?

[Journal of Biomechanics 33 (2000) 953-960]

A
  1. Deepens the acetabulum and extends the coverage of the femoral head
  2. Contributes to a negative pressure vacuum effect which adds stability to the hip joint
    * Greater force required to distract joint
  3. Provides a seal against fluid flow in and out of the intra-articular space enhancing lubrication mechanisms
    * Encapsulates the fluid in the joint
  4. Limits the rate of fluid expression from the cartilage during loading which enhances the cartilages ability to carry load and limit stresses on the cartilage
How well did you know this?
1
Not at all
2
3
4
5
Perfectly
10
Q

What is the Seldes classification of hip labral tears?

[Clin Orthop Relat Res. 2001 Jan;(382):232-40]

A

Type 1 – “Detachment”

  • Detachment of the labrum from the articular hyaline cartilage at the transition zone

Type 2 – “Intrasubstance”

  • One or more cleavage planes of variable depth within the substance of the labrum
How well did you know this?
1
Not at all
2
3
4
5
Perfectly
11
Q

What are the causes of hip labral tears?

[J Am Acad Orthop Surg 2017;25:e53-e62]

A
  1. Trauma
  2. FAI
  3. Dysplasia
  4. Hip hypermobility/capsular laxity
  5. Degeneration
How well did you know this?
1
Not at all
2
3
4
5
Perfectly
12
Q

How can you classify damage to the ligamentum teres, labrum and articular cartilage during hip arthroscopy?

[J Am Acad Orthop Surg 2017;25:e53-e62]

A
  1. Domb classification of ligamentum teres tears
  • Grade 0 = No tear
  • Grade 1 = <50% tear
  • Grade 2 = >50% tear
  • Grade 3 = 100% tear
  1. Seldes Classification of labral tears
  • Grade 1 - chondrolabral junction tear
  • Grade 2 - intrasubstance tear
  1. ALAD (acetabular labrum articular disruption) Classification
  • Grade 1 - softening of the adjacent cartilage
  • Grade 2 - early peel of cartilage
    • Carpet delamination
  • Grade 3 - large flap of cartilage
  • Grade 4 - loss of cartilage
    4. Outerbridge classification
  • Grade 0 - normal cartilage
  • Grade 1 - cartilage with softening and swelling
  • Grade 2 - partial thickness defect with fissures on the surface that do not reach subchondral bone or exceed 1.5cm in diameter
  • Grade 3 - fissuring to the level of the subchondral bone in an area with a diameter larger than 1.5cm
  • Grade 4 - exposed subchondral bone
How well did you know this?
1
Not at all
2
3
4
5
Perfectly
13
Q

Describe the decision making algorithm when considering arthroscopic labral debridement vs. repair vs. reconstruction

[J Am Acad Orthop Surg 2017;25:e53-e62]

A
  1. Stable torn labrum
  • Acetabuloplasty not needed = selective debridement
  • Acetabuloplasty needed = repair
  1. Unstable torn labrum
  • Viable tissue = repair
  • Nonviable tissue, young patient = reconstruction
  • Poor vascularity or advanced age = selective debridement
  1. Mostly calcified torn labrum
  • Advanced age = selective debridement
  • Young = reconstruction
How well did you know this?
1
Not at all
2
3
4
5
Perfectly
14
Q

What are the 3 main types of FAI?

A
  1. Cam impingement – femoral based abnormality
  2. Pincer impingement – acetabular based abnormality
  3. Combined/mixed-type
How well did you know this?
1
Not at all
2
3
4
5
Perfectly
15
Q

What are the features of a cam-lesion?

A
  1. Aspherical femoral head
  2. Reduced head-neck offset
  3. Characteristic ‘bump’ at the head-neck junction
  4. Pistol grip deformity
How well did you know this?
1
Not at all
2
3
4
5
Perfectly
16
Q

Where is the typical cam-lesion located?

[J Am Acad Orthop Surg 2013; 21(suppl 1):S20-S26]

A

Anterosuperior head-neck junction

How well did you know this?
1
Not at all
2
3
4
5
Perfectly
17
Q

What are the features of the pincer-lesion?

[J Bone Joint Surg Am. 2013;95:82-92]

A
  1. Global overcoverage
    * Coxa profunda, coxa protrusio
  2. Focal overcoverage
    * Cephalad retroversion
  3. Acetabular retroversion
How well did you know this?
1
Not at all
2
3
4
5
Perfectly
18
Q

What femur orientation contributes to FAI – anteversion or retroversion?

A

Femoral retroversion

How well did you know this?
1
Not at all
2
3
4
5
Perfectly
19
Q

What radiographs and radiographic findings are important in assessing FAI?

[DeLee & Drez’s, 2015]

A
  1. Radiographic views
  • AP pelvis
  • Lateral view
    • Frog-leg lateral, Dunn views, cross-table lateral
  • False profile view
  1. Signs of pincer-lesion
  • Crossover sign [AAOS comprehensive review 2, 2014]
    • Normally the anterior lip of the acetebulum lies medial to the posterior lip and converge at the superolateral aspect of the acetabulum
    • With retroversion the anterior lip proximally lies lateral to the posterior lip and distally lies medially creating the crossover sign
  • Prominent ischial spine sign
    • Normally the ischial spine is hidden behind the acetabulum, if it appears more prominent it indicates acetabular retroversion
  • Posterior wall sign
    • Posterior rim of the acetabulum lies medial to the center of rotation of the femoral head indicating retroversion
  • Lateral center edge angle
    • Formed by a vertical line and a line connecting the femoral head center with the lateral edge of the acetabulum
    • LCEA >40 suggests pincer-lesion
  • Os acetabulum
    3. Signs of cam-lesion [JAAOS 2013;21(suppl 1):S20-S26]
  • Alpha angle
    • Formed by a line along the axis of the femoral neck and a line from the center of the femoral head to the point where the head diverges outside the circle
    • >50 degrees is associated with FAI
  • Head-neck offset and offset ratio
    • Based on a lateral view, a line parallel to the long axis of the femoral neck is drawn along the anterior femoral neck and second line along the anterior aspect of the femoral head
    • The distance between the two is the head neck offset
      • <8mm likely represents cam-lesion
    • Offset ratio is the distance between the two lines divided by the diameter of the femoral head
      • <0.17 likely represents cam-lesion
How well did you know this?
1
Not at all
2
3
4
5
Perfectly
20
Q

What radiographic view best demonstrates the maximal CAM deformity?

[JAAOS 2013;21(suppl 1):S20-S26]

A

45° Dunn view

How well did you know this?
1
Not at all
2
3
4
5
Perfectly
21
Q

What radiographic view best demonstrates the anterior CAM deformity?

[JAAOS 2013;21(suppl 1):S20-S26]

A

Cross table lateral and frog leg lateral

How well did you know this?
1
Not at all
2
3
4
5
Perfectly
22
Q

What special tests should be performed during the physical exam for FAI?

[J Am Acad Orthop Surg 2013; 21(suppl 1):S16-S19]

A
  1. Impingement test (FADIR)
    * With the hip at 90° of hip flexion the hip is internally rotated and adducted
  2. Posterior impingement test
    * Hip extension combined with external rotation
  3. Log roll test
  4. Resisted hip flexion test
  5. FABER
How well did you know this?
1
Not at all
2
3
4
5
Perfectly
23
Q

Associated injuries in cam impingement include?

[Orthop Clin N Am 44 (2013) 575–589]

A
  1. Labral detachment from the acetabular rim
  2. Cartilage delamination (full and partial-thickness)
    * Deeper compared to peripheral cartilage delamination in pincer impingement
How well did you know this?
1
Not at all
2
3
4
5
Perfectly
24
Q

Associated injuries in pincer impingement include?

[Orthop Clin N Am 44 (2013) 575–589]

A
  1. Labral pathology
  2. Peripheral cartilage delamination
  3. Contracoup chondrolabral lesions in the posterior acetabulum
    * Due to anterior levering of the femur causing posterior shear
How well did you know this?
1
Not at all
2
3
4
5
Perfectly
25
Q

What are the risk factors for femoral neck fracture after hip arthroscopy for FAI?

[J Hip Preserv Surg. 2017 Jan; 4(1): 9–17]

A
  1. Femoral osteochondroplasty
  2. Early WB
  3. Resection depth 10-33% the width of the femoral neck
  4. Increased age
How well did you know this?
1
Not at all
2
3
4
5
Perfectly
26
Q

What percent chance exists to have a recurrent dislocation if a patient has had 2 prior patella dislocations?

A

50%

How well did you know this?
1
Not at all
2
3
4
5
Perfectly
27
Q

What are the factors that contribute to patella stability?

[JAAOS 2011;19:8-16]

A
  1. Osseous anatomy
  • Trochlear groove
    • Most important stabilizer at flexion >30°
    • Height and slope of the lateral trochlear facet provides the primary resistance
  1. Static stabilizers
  • MPFL and medial patellotibial ligament
    • Most important in full extension when patella is not engaged in trochlea (between 0-30)
    • Function to guide the patella into the groove from 0-20° flexion
      1. Dynamic stabilizers
  • VMO
    • More variable contribution to patella instability
How well did you know this?
1
Not at all
2
3
4
5
Perfectly
28
Q

What are risk factors for patellar instability?

[JAAOS 2011;19:8-16]

A
  1. Unfavourable bone anatomy
  • Trochlea dysplasia (shallow, flattened trochlea groove)
    • Seen in <2% of the general population but in 85% of recurrent patellar instability
  • Patella alta
  • Increased TT-TG distance
  • Increased Q angle
  • Lower extremity rotational malalignment
    • femoral anteversion +/- external tibial torsion
      1. Dynamic stabilizer imbalance
  • ITB
    • Increased tension causes lateral patellar tracking
  • VMO/VLO
    • Imbalance in strength can lead to instability
  1. Incompetent static stabilizers
    * Generalized ligamentous laxity
How well did you know this?
1
Not at all
2
3
4
5
Perfectly
29
Q

Where anatomically is the MPFL typically injured?

[JBJS 2016;98:417-27]

A

Femoral origin

How well did you know this?
1
Not at all
2
3
4
5
Perfectly
30
Q

What is the classification of trochlear dysplasia?

[JAAOS 2011;19:8-16]

A

Dejour classification

  • Type A = shallow
    • Radiographic findings = crossing sign
  • Type B = flat
    • Radiographic findings = crossing sign, supratrochlear spur
  • Type C = lateral convexity/medial hypoplasia
    • Radiographic findings = crossing sign, double contour sign
  • Type D = cliff
    • Radiographic findings = crossing sign, double contour sign, supratrochlear spur
How well did you know this?
1
Not at all
2
3
4
5
Perfectly
31
Q

Why is the patella more unstable (and thus more prone to dislocation) in knee extension rather than flexion?

[J Bone Joint Surg Am. 2008;90:2751-62]

A
  1. Q-angle is greatest in knee extension
  2. Lowest quadriceps and patellar tendon tension
    * Low posterior directed force
  3. Patella disengages from the trochlear groove
How well did you know this?
1
Not at all
2
3
4
5
Perfectly
32
Q

Describe the anatomy of the MPFL?

A

MPFL

  • Femoral attachment
    • Center of attachment is between the medial epicondyle and the adductor tubercle
  • Patella attachment
    • 7.4mm anterior to a tangent to the posterior patellar cortical line and 5.4mm distal to the proximal edge of the articular surface of the patella
    • Located at the junction of the proximal 1/3 and distal 2/3
    • Encompasses 33% of the total length of the patella [JAAOS 2014;22:175-182]
How well did you know this?
1
Not at all
2
3
4
5
Perfectly
33
Q

What is the radiographic landmark of the femoral insertion of the MPFL called and where is it located?

[Orthop Clin N Am 46 (2015) 147–157]

A
  1. Schottle point
  • 1 mm anterior to the posterior cortex extension line
  • 2.5 mm distal to the posterior origin of the medial femoral condyle
  • Proximal to the level of the posterior point of the Blumensaat line
  1. Stephens normalized dimensions [JAAOS 2014;22:175-182]
  • If AP dimension of medial femoral condyle is 100%, MPFL attachment is:
    • 40% from posterior
    • 50% from distal
    • 60% from anterior
How well did you know this?
1
Not at all
2
3
4
5
Perfectly
34
Q

What are the physical examination findings in patellar instability?

[JAAOS 2018;26:429-439]

A
  1. J sign
  • Knee is actively brought from flexion to extension
  • Positive = Patella demonstrates a sudden, exaggerated lateral deviation after it fully exits the trochlear groove in full extension
    • Indicates either trochlear dysplasia or patella alta
  1. Moving patellar apprehension test
  • Most sensitive and specific
  • Part 1:
    • Knee held in full extension and the patella is manually translated laterally with the thumb
    • Knee is then flexed to 90o and then brought back to full extension while the lateral force on the patella is maintained
    • Positive:
      • Patient orally expresses apprehension and may activate his or her quadriceps in response to apprehension
  • Part 2:
    • Knee is started in full extension, brought to 90o of flexion, and then back to full extension while the index finger is used to translate the patella medially
    • Positive:
      • Patient experiences no apprehension and allows free flexion and extension of the knee
  1. Lateral patellar translation
    * Normal = Lateral translation of the patella should be ¼ to ½ the width of the patella
How well did you know this?
1
Not at all
2
3
4
5
Perfectly
35
Q

What are radiographic features to assess for in patellar instability?

[Orthop Clin N Am 46 (2015) 147–157] [J Bone Joint Surg Am. 2008;90:2751-62] [JAAOS 2011;19:8-16] )

A
  1. Patella alta
  • Insall-Salvati ratio (normal = 0.8-1.2)
  • Caton-Deschampe ratio (normal = 0.6-1.3)
  • Blackburn-Peel ratio (normal = 0.5-1.0)
  • Blumensaat’s line should extend to the inferior pole of the patella at 30° of knee flexion
    2. Trochlear dysplasia
  • Signs on lateral radiograph include:
    • Normally, floor of the trochlea should not pass anterior to a line extended along the anterior femoral cortex
    • Crossing sign:
      • A line represented by the deepest part of the trochlear groove crossing the anterior aspect of the condyles
    • Supratrochlear spur/prominence
      • Extension of the trochlear groove above the projection of the anterior cortex of the femur
      • Measured as the distance between the anterior femoral cortex and most anterior point of the trochlear floor
        • Abnormal = >4mm
    • Double contour
  • Signs on the skyline view
    • Sulcus angle
      • Measured from the highest point on the condyles to the lowest point in the intercondylar sulcus
      • Normal = 138 ± 6°
      • Abnormal = >145°
        • Indicates trochlear dysplasia
          3. Lateral patellar tilt/subluxation
  • Assess on skyline/Merchant view
How well did you know this?
1
Not at all
2
3
4
5
Perfectly
36
Q

What radiograph is superior for evaluating trochlear anatomy?

[JAAOS 2018;26:429-439]

A

Laurin radiograph

  • Knee flexed only 20°
  • Imaging beam directed from inferior to superior

***45° sunrise view does not visualize the proximal trochlear groove (may miss supratrochlear spur)

How well did you know this?
1
Not at all
2
3
4
5
Perfectly
37
Q

What is the role of CT in assessing patellar instability?

[Orthop Clin N Am 46 (2015) 147–157] [JAAOS 2018;26:429-439]

A

Used to assess the tibial tubercle-trochlear groove distance

  • Measured by taking a line perpendicular to a line tangential to the posterior femoral condyles through the deepest part of the trochlear groove and through the apex of the tibial tubercle
    • Distance between these 2 lines is defined as the TT-TG distance
How well did you know this?
1
Not at all
2
3
4
5
Perfectly
38
Q

What TT-TG distance is associated with patellar instability?

A

>20mm (average normal is 9mm)

  • 10-15mm = normal
    • MPFL reconstruction is usually sufficient
  • 15-20mm = gray zone
    • Consider bony reconstruction in addition to MPFL reconstuction
  • >20mm = abnormal
    • MPFL reconstruction unlikely to be sufficient
    • Needs bony reconstruction in addition to MPFL

***NOTE: 20mm is a guideline and not a rule; MRI values may be less

How well did you know this?
1
Not at all
2
3
4
5
Perfectly
39
Q

What measurement may more accurately describe tibial tubercle lateralization than TT-TG?

[JAAOS 2018;26:429-439]

A

Tibial tubercle – posterior cruciate ligament distance

  • Normal <24mm
  • Both are tibial reference points
    • Not affected by knee rotation like TT-TG
How well did you know this?
1
Not at all
2
3
4
5
Perfectly
40
Q

What MRI findings are consistent with patellar instability?

[J Bone Joint Surg Am.2008;90:2751-62][Sports Med Arthrosc Rev 2016;24:44–49)]

A
  1. Bone bruising
    * Edema on lateral aspect of lateral femoral condyle and inferomedial patella
  2. Cartilage damage
    * Osteochondral fracture of the medial patellar facet or lateral trochlear ridge
  3. MPFL damage
  4. VMO injury
How well did you know this?
1
Not at all
2
3
4
5
Perfectly
41
Q

What is the treatment of choice for first-time patella dislocation?

[JBJS 2016;98:417-27]

A

Nonoperative treatment

  • Bracing
  • ROM
  • Strengthening
  • Gradual return to play
How well did you know this?
1
Not at all
2
3
4
5
Perfectly
42
Q

What is the standard surgical approach in patient with chronic lateral patellar instability with at least 2 documented patellar dislocations?

[JAAOS 2014;22:175-182]

A

Anatomic MPFL reconstruction

  • Miniopen technique
  • Using a graft stronger than the native MPFL
    • Semi-T or gracilis
    • Compensates for the uncorrected predisposing patellar instability factors
How well did you know this?
1
Not at all
2
3
4
5
Perfectly
43
Q

What is the ideal indication for an MPFL reconstruction?

[JAAOS 2014;22:175-182]

A
  1. Recurrent lateral patellar dislocation
  2. TT-TG distance <20 mm
  3. Positive apprehension test until 30° of knee flexion
  4. Caton-Deschamps index of <1.2
  5. Normal trochlea or Dejour type A dysplasia
How well did you know this?
1
Not at all
2
3
4
5
Perfectly
44
Q

What are the errors in femoral tunnel position for MPFL reconstruction and their implications?

[JAAOS 2014;22:175-182]

A
  1. Too proximal
  • Graft lax in extension and tight in flexion
  • Clinically:
    • Anterior knee pain
    • Loss of flexion
    • Graft stretch and failure
  1. Too distal
  • Graft tight in extension and lax in flexion
  • Clinically:
    • Extension lag
How well did you know this?
1
Not at all
2
3
4
5
Perfectly
45
Q

Where is the MPFL graft typically passed between the patella and femur?

A

Between layer 2 (medial patellar retinaculum) and layer 3 (capsule)

How well did you know this?
1
Not at all
2
3
4
5
Perfectly
46
Q

Is the native MPFL isometric?

[JAAOS 2014;22:175-182]

A

The MPFL is nonisometric over the complete ROM

  • Isometric from 0-30o
  • Tight in extension, Lax in flexion
How well did you know this?
1
Not at all
2
3
4
5
Perfectly
47
Q

How should the MPFL graft function after reconstruction?

[JAAOS 2014;22:175-182]

A
  1. The ROM after MPFL graft fixation should be complete
  2. The MPFL graft should be isometric from 0-30°
  3. The MPFL graft should tighten in extension and be lax in flexion
  4. There should be a good endpoint to lateral patellar translation from 0-30°
  5. The MPFL should only tighten on lateral patellar translation
  6. Should permit 1cm of lateral translation or the equivalent of two quadrants of lateral deviation with a firm end point
How well did you know this?
1
Not at all
2
3
4
5
Perfectly
48
Q

What are the most common complications following MPFL reconstruction?

[JBJS 2016;98:417-27]

A
  1. Recurrent apprehension
  2. Arthrofibrosis
  3. Pain
  4. Clinical failure
  5. Patellar fracture
How well did you know this?
1
Not at all
2
3
4
5
Perfectly
49
Q

What is the redislocation rate following MPFL reconstruction?

[JAAOS 2018;26:429-439]

A

<10%

How well did you know this?
1
Not at all
2
3
4
5
Perfectly
50
Q

What are the indications and contraindications for a tibial tubercle transfer in the setting of patellar instability?

[JAAOS 2018;26:429-439]

A

Indications:

  • TT-TG >20mm (measured on CT)
  • High TT-PCL distance (>24mm)
  • Caton-Deschamp ratio >1.2

Contraindications [JBJS 2016;98:417-27]

  • Open tibial apophysis
    • Causes growth arrest and recurvatum
  • Medial dislocations
  • Patellofemoral OA of the proximal and medial facets
How well did you know this?
1
Not at all
2
3
4
5
Perfectly
51
Q

What is the postoperative goal TT-TG after tibial tubercle transfer for patellar instability?

A
  1. 10mm [JAAOS 2018;26:429-439]
  2. 9-15mm [JBJS 2016;98:417-27]
How well did you know this?
1
Not at all
2
3
4
5
Perfectly
52
Q

What is the main complication following tibial tubercle transfer for patellar instability?

[JBJS 2016;98:417-27]

A

Symptomatic hardware

How well did you know this?
1
Not at all
2
3
4
5
Perfectly
53
Q

In the presence of trochlear dysplasia what is the recommended treatment for patellar instability?

[JAAOS 2011;19:8-16]

A
  1. First-line
  • Procedures to address associated factors rather than trochleoplasty
    • TT-TG and patella height normal = MPFL reconstruction
    • TT-TG >20mm = tibial tubercle transfer or femoral derotation osteotomy
      1. Second-line
  • Trochleoplasty
    • Involves reshaping the trochlea by deepening the groove by removing subchondral bone and impacting the overlying catilage into the defect
    • Indications:
      • High-grade trochlear dysplasia (where other options won’t provide stability) and salvage situations
How well did you know this?
1
Not at all
2
3
4
5
Perfectly
54
Q

What type of trochlea according to the Dejour classification is most likely to benefit from a trochleoplasty?

[JAAOS 2018;26:429-439]

A

Type B and D

  • Both have supratrochlear spurs – tend to displace the patella laterally
How well did you know this?
1
Not at all
2
3
4
5
Perfectly
55
Q

What are the contraindications for trochleoplasty?

[JAAOS 2018;26:429-439]

A
  1. Open physes
  2. Advanced patellofemoral arthritis
How well did you know this?
1
Not at all
2
3
4
5
Perfectly
56
Q

What are the indications for tibial tubercle osteotomy (distal realignment procedures)?

[Clin Sports Med 33 (2014) 517–530]

A
  1. Recurrent patella instability if
  • Skeletally mature
    • Proximal tibia and tibial tubercle physis closed
  • TT-TG >15
  • Caton-Deschamps >1.2
  1. Unload focal patella cartilage lesions
  • Without cartilage resurfacing for lateral or distal lesions
  • With cartilage resurfacing for central or medial lesions
  1. Isolated lateral patellofemoral compression, tilt or overload in patients who fail lateral release
How well did you know this?
1
Not at all
2
3
4
5
Perfectly
57
Q

What are contraindications for TTO?

[Clin Sports Med 33 (2014) 517–530] [AJSM 2014; 42(8): 2006)

A
  1. Skeletally immature
  2. Medial patellar subluxation/dislocation
  3. Diffuse patellofemoral arthrosis
  4. Smoking
How well did you know this?
1
Not at all
2
3
4
5
Perfectly
58
Q

What are the types of TTOs?

[Clin Sports Med 33 (2014) 517–530]

A
  1. Medialization (Elmslie-Trillat procedure)
  • Reduces the TT-TG
  • Decreases pressure on lateral facet
  • Amount of medialization is limited by the amount of bony contact available
  1. Anterior transfer (Maquet)
  • Goal is to reduce patellofemoral contact pressures
  • Transfers the contact forces from distal to proximal patella
  • Requires allograft or autograft bone block
  • Not routinely done
    • Anteriorization usually combined with medialization
  • Can be indicated for unloading large distal patellar chondral lesions, bipolar kissing lesions or arthritis in setting of normal TT-TG
    3. Distalization
  • Indications:
    • Caton-Deschamps or Insall-Salvati >1.2 (usually >1.4)
  • Rarely done in isolation
    • Usually combined multiplanar osteotomy
  • Caution with overdistalization which can limit knee flexion
  1. Anteromedialization (Fulkerson)
  • The amount of anteriorization and medialization is determined by the obliquity of the osteotomy
    • Increased obliquity [A-P] = increased anteriorization
    • With a constant anteriorization of 15mm: [AJSM 2014; 42(8): 2006)
      • 60° slope creates ~9mm of medialization
      • 45° slope creates ~15mm of medialization
  • Advantages:
    • Preserves extensor mechanism
    • Large surface for bone healing
    • Ability to place multiple screws
    • Multiplanar adjustments
    • Early ROM
  • Disadvantages
    • Does not address incompetent MPFL
    • Hardware irritation
    • Potential NV injury
    • Increased medial PF contact pressures
    • Delayed union/nonunion
    • Cannot perform in skeletally immature
How well did you know this?
1
Not at all
2
3
4
5
Perfectly
59
Q

What are the complications associated with TTO?

[Clin Sports Med 33 (2014) 517–530][AJSM 2014; 42(8): 2006)

A
  1. Hardware irritation (50% require removal)
  2. Nonunion/delayed union
  3. Compartment syndrome
  4. Anterior tibial artery injury
  5. Deep peroneal injury
  6. Infection
  7. DVT
  8. Medial patellar instability
  9. Fracture of tubercle
  10. Fracture of proximal tibia
  11. Skin necrosis
How well did you know this?
1
Not at all
2
3
4
5
Perfectly
60
Q

What is the etiology of ACL tears?

[J Am Acad Orthop Surg 2013;21:41-50]

A
  1. Noncontact injuries (70%)
    * Most commonly a deceleration event and change in direction with a planted foot (cutting maneuver)
  2. Trauma (30%)
How well did you know this?
1
Not at all
2
3
4
5
Perfectly
61
Q

What are the risk factors for an ACL tear?

[J Am Acad Orthop Surg 2013;21:41-50]

A
  1. Female sex
  2. Increased friction and shoe-playing surface interface (eg. cleats)
  3. Notch stenosis
  4. Increased posterior tibial slope
  5. Increased Q-angle
  6. Smaller ACL
  7. Increased quadriceps to hamstring strength
  8. Poor landing position
    * Erect, hips adducted/internally rotated, knee relatively extended/valgus, tibia externally rotated
How well did you know this?
1
Not at all
2
3
4
5
Perfectly
62
Q

What factors make females 2-8x more likely than male athletes to sustain an ACL injury?

[JAAOS 2013;21:41-50]

A
  1. Increased Q angle
  2. Notch stenosis
  3. Smaller ACL
  4. Increased quadriceps to hamstring strength and recruitment ratio
  5. Poor landing position
  6. Others:
  • Increased medial posterior tibial slope
  • Hormonal factors
  • Increased generalized ligamentous laxit
  • Increased knee laxity
How well did you know this?
1
Not at all
2
3
4
5
Perfectly
63
Q

What are the indications for ACL reconstruction?

[AAOS comprehensive review 2, 2014]

A
  1. Athletes who perform cutting and pivoting sports
  2. High demand occupations (eg. police or military)
  3. High risk occupations (eg. firefighter)
  4. Recurrent instability
How well did you know this?
1
Not at all
2
3
4
5
Perfectly
64
Q

What are indications for nonoperative management of ACL injuries?

[AAOS comprehensive review 2, 2014]

A
  1. Low physical demand patients
  2. Older age
  3. Advanced osteoarthritis
  4. Patients unwilling or unable to comply with postoperative rehab
How well did you know this?
1
Not at all
2
3
4
5
Perfectly
65
Q

What radiographic features would suggest a possible ACL injury (4)?

A
  1. Effusion
  2. Segond fracture
  3. Tibial tubercle avulsion
  4. Lateral femoral notch sign/deep sulcus sign
How well did you know this?
1
Not at all
2
3
4
5
Perfectly
66
Q

What are the MRI findings to support the diagnosis of an ACL tear?

A
  1. Direct
  • Fibre discontinuity
  • Increased signal intensity
  • Abnormal orientation
  • ‘Empty notch sign’ (chronic)
    2. Indirect
  • Bone bruises
  • Anterior tibial translation
  • PCL buckling (reduced PCL angle)
  • Uncovering of the posterior horn of the lateral meniscus
How well did you know this?
1
Not at all
2
3
4
5
Perfectly
67
Q

What is the collagen composition of the ACL?

[World J Orthop 2015,18;6(2):252-262]

A

Type I – 90%

Type III – 10%

How well did you know this?
1
Not at all
2
3
4
5
Perfectly
68
Q

What are the bundles of the ACL and what are their function?

[Sports Med Arthrosc Rev 2010;18:27–32] [Clin Sports Med 36 (2017) 9–23]

A

Bundle named based on tibial insertion

  • Anteromedial
    • Main contributor to AP stability
    • Taught in flexion, relaxed in extension
  • Posterolateral
    • Main contributor to rotational stability
    • Taught in extension, relaxed in flexion
How well did you know this?
1
Not at all
2
3
4
5
Perfectly
69
Q

Describe the femoral insertion of the ACL?

A

Medial wall of the lateral femoral condyle

  • Posterior to the resident’s ridge (lateral intercondylar ridge)
  • Lateral bifurcate ridge separating the AM from the PL bundle
    • AM bundle superior and PL bundle is inferior to bifurcate ridge
How well did you know this?
1
Not at all
2
3
4
5
Perfectly
70
Q

What are the arthroscopic landmarks to determine the centre of the ACL femoral footprint (single bundle technique)?

[JAAOS 2016;24:443-454]

A
  1. 8mm anterior to the posterior articular margin of the lateral femoral condyle
  2. 1.7mm proximal to the bifurcate ridge
How well did you know this?
1
Not at all
2
3
4
5
Perfectly
71
Q

How do you landmark the ACL femoral footprint using fluoroscopy?

[JAAOS 2016;24:443-454]

A

For single-bundle reconstruction:

  • Center of the femoral footprint can be referenced on a grid using the Blumensaat line
  • 28% of the distance from proximal to distal
  • 34% posterior to the Blumensaat line
How well did you know this?
1
Not at all
2
3
4
5
Perfectly
72
Q

Describe the tibial insertion of the ACL?

[Clin Sports Med 36 (2017) 9–23]

A

Broad insertion anterior and between the medial and lateral tibial spines

  • AM bundle located more anterior and medial
  • PL bundle located more posterior and lateral
How well did you know this?
1
Not at all
2
3
4
5
Perfectly
73
Q

What are the arthroscopic landmarks to determine the centre of the ACL tibial footprint (single bundle technique)?

[JAAOS 2016;24:443-454]

A
  1. 9mm posterior to the posterior edge of the intermeniscal ligament
  2. 5mm anterior to the peak of the medial tibial spine
  3. Midway between the medial and lateral tibial spines in the coronal plane

***Note: not recommended to use the lateral meniscus as a reference as the anterior horn insertion is variable

  • If used the centre of the footprint is posterior and medial to the anterior insertion
How well did you know this?
1
Not at all
2
3
4
5
Perfectly
74
Q

How do you landmark the ACL tibial footprint using fluoroscopy?

[JAAOS 2016;24:443-454]

A
  1. 43% of the distance anterior to posterior of the midsagittal tibial diameter
  2. 51% medial to lateral on the AP view
How well did you know this?
1
Not at all
2
3
4
5
Perfectly
75
Q

On a postoperative radiograph, what is the ideal ACL tibial tunnel position and femoral tunnel position?

[Skeletal Radiol (2013) 42:1489–1500]

A
  1. Lateral
  • Anterior wall of the tibial tunnel should be posterior to a line extended along Blumensaat’s line with knee in extension
  • No part of the femoral tunnel should intersect Blumensaat’s line
  1. AP
  • Angle between the femoral anatomical axis and the femoral tunnel ideally is 39°
    • ≤17° is associated with rotational instability
How well did you know this?
1
Not at all
2
3
4
5
Perfectly
76
Q

Describe the relationship of the ACL AM to the PL bundle in extension compared to flexion?

[World J Orthop 2015,18;6(2):252-262]

A
  1. Extension
  • Femoral insertions are vertical (AM above PL)
  • Bundles are parallel
  • PL bundle is tight and AM bundle is relaxed
  1. Flexion
  • Femoral insertions are horizontal (AM posterior to PL)
  • Bundles are crossed
  • PL bundle is relaxed and AM bundle is tight
How well did you know this?
1
Not at all
2
3
4
5
Perfectly
77
Q

What is the surgical terminology used when describing the ACL femoral tunnel placement?

A

With knee in flexion as occurs during surgery:

  • Shallow/deep (A-P)
  • High/low (superior-inferior)
How well did you know this?
1
Not at all
2
3
4
5
Perfectly
78
Q

Is the native ACL isometric?

[World J Orthop 2015,18;6(2):252-262]

A

No

  • “Isometry in ACL does not exist as there is no one point on femur that maintains a fixed distance from a single point on the tibia during the range of motion of the knee.”
How well did you know this?
1
Not at all
2
3
4
5
Perfectly
79
Q

What is the rational for isometric ACL graft placement?

[World J Orthop 2015,18;6(2):252-262]

A
  1. An isometric graft avoids changes in graft length and tension during knee flexion and extension which avoids graft failure by overstretching
  2. Downside is it results in a more vertical graft which is not as effective in controlling rotation
  3. Isometric point on the femoral side is high in the notch and posterior
How well did you know this?
1
Not at all
2
3
4
5
Perfectly
80
Q

What clock position is thought to be optimal for the ACL femoral tunnel to achieve graft isometry?

[World J Orthop 2015,18;6(2):252-262]

A

11 o’clock (right) and 1 o’clock (left)

  • Apex of the notch represented by 12 o’clock

***NOTE – changing the position to 10 o’clock (right) or 2 o’clock (left) improves the rotational stability

  • Graft becomes less vertical
How well did you know this?
1
Not at all
2
3
4
5
Perfectly
81
Q

What is the blood supply of the ACL?

[Clin J Sport Med 2012;22:349–355]

A
  1. Primary – middle genicular artery
  2. Secondary – inferomedial and inferolateral genicular arteries
How well did you know this?
1
Not at all
2
3
4
5
Perfectly
82
Q

What is the innervation of the ACL?

[Clin Sports Med 36 (2017) 9–23]

A

Branches of the tibial nerve

How well did you know this?
1
Not at all
2
3
4
5
Perfectly
83
Q

What is the average length of the native ACL?

[World J Orthop 2015,18;6(2):252-262]

A

33mm

How well did you know this?
1
Not at all
2
3
4
5
Perfectly
84
Q

What is a cyclop lesion (in context of ACL)?

A

Focal nodule of fibrous tissue sitting in the intercondylar notch anterior to the reconstructed ACL

  • Significance:
    • Limits complete knee extension due to intercondylar notch impingement
How well did you know this?
1
Not at all
2
3
4
5
Perfectly
85
Q

With an ACL injury, what is the typical bone bruises identified on MRI and what is their significance?

[AAOS comprehensive review 2, 2014][Sports Med Arthrosc Rev 2016;24:44–49]

A
  1. Anterolateral femoral condyle (sulcus terminalis) and posterolateral tibial plateau (“kissing lesion”)
  2. Compared to those without bone bruises:
  • Protracted clinical recovery
  • Greater effusions and pain scores
  • Slower return of motion
How well did you know this?
1
Not at all
2
3
4
5
Perfectly
86
Q

What is the most common meniscus injured in the acute vs. chronic ACL deficient knee?

[J Am Acad Orthop Surg 2013;21:204-213]

A
  1. Acute ACL injury – lateral meniscus tear
  2. Chronic ACL injury – medial meniscus tear
How well did you know this?
1
Not at all
2
3
4
5
Perfectly
87
Q

What are graft options for ACL reconstruction?

A
  1. Hamstring autograft
  2. Bone-patellar-bone autograft
  3. Quadriceps tendon autograft
  4. Allograft
How well did you know this?
1
Not at all
2
3
4
5
Perfectly
88
Q

Which ACL graft has the greatest strength and stiffness?

A

Quadrupled-strand hamstring graft

How well did you know this?
1
Not at all
2
3
4
5
Perfectly
89
Q

What ACL graft size diameter (mm) has been shown to decrease failure rates?

[Arthroscopy 2014 Jul;30(7):882-90]

A

8mm

  • “Quadrupled-strand hamstring autograft with a diameter equal to or larger than 8 mm decreases failure rates. Grafts larger than 8 mm were found to provide a protective effect in patients aged younger than 20 years, a group identified to be at increased risk of failure”
How well did you know this?
1
Not at all
2
3
4
5
Perfectly
90
Q

What is the concern with ACL grafts that are too large?

[JBJS 2017;99:438-45]

A

Impingement of the graft on the roof and PCL

How well did you know this?
1
Not at all
2
3
4
5
Perfectly
91
Q

When selecting an interference screw for ACL soft tissue grafts, how can you increase the fixation strength?

[JBJS 2017;99:438-45]

A

Increase screw length or diameter

How well did you know this?
1
Not at all
2
3
4
5
Perfectly
92
Q

What is the general recommendation for notchplasty during ACL reconstruction?

[JBJS 2017;99:438-45]

A

Notchplasty is not recommended during ACL reconstruction

  • Smaller intercondylar notch dimensions are not associated with ACL graft failure
How well did you know this?
1
Not at all
2
3
4
5
Perfectly
93
Q

What are the cons of using an ACL BTB autograft?

[Clin J Sport Med 2012;22:349–355] [DeLee & Drez’s, 2015]

A
  1. Anterior knee pain
  2. Loss of sensation
  3. Patellar fracture
  4. Inferior patellar contracture
  5. Patellar tendon rupture
  6. Quadriceps weakness
94
Q

What is one way to increase the strength of an ACL BTB autograft?

[JBJS 2017;99:438-45]

A

Twisting the graft

  • “Twisting the graft by 90° increased the ultimate strength by 30%”
95
Q

What portion of the ACL BTB graft is the strongest and stiffest – medial, central or lateral 1/3?

[JBJS 2017;99:438-45]

A

Central 1/3

96
Q

What are two ways to manage graft tunnel mismatch when an ACL BTB graft is too long?

[JBJS 2017;99:438-45]

A
  1. Rotate the graft
  • Based on rotation from the proximal (patellar) end
  • External rotation has been shown to more significantly reduce graft length compared with internal rotation
    • Shortening of 25% at 630° external rotation
  1. Single bone plug
    * Proximal patellar tendon is removed from patella without a bone plug
97
Q

What are the cons of using hamstring autograft for ACL reconstruction?

[Clin J Sport Med 2012;22:349–355]

A
  1. Decreased hamstring strength and endurance
  2. Difficult to harvest due to variable graft length and diameter
98
Q

What are the advantages and disadvantages of allograft for ACL reconstruction?

[Clin J Sport Med 2012;22:349–355]

A

Advantages

  • Avoid donor-site morbidity
  • Reduced operative time
  • Availability of larger grafts
  • Superior cosmesis
  • Possibility for multiple ligament reconstructions

Disadvantages

  • Delayed graft incorporation
  • Disease transmission
  • Potential immune reactions
  • Altered mechanical properties caused by sterilization
  • Cost of the allograft
99
Q

Which ACL graft, hamstring or BTB, shows greater migration of osseus fixation and graft stretching?

[JBJS 2017;99:438-45]

A

Hamstring

100
Q

What are the 3 main techniques for ACL reconstruction with respect to femoral tunnel preparation?

A
  1. Anteromedial
  2. Transtibial
  3. Outside-in
101
Q

Which ACL reconstruction technique results in a more anatomic femoral tunnel position?

[World J Orthop 2015,18;6(2):252-262]

A

Anteromedial portal technique

102
Q

What is the clinical difference between single and double bundle ACL reconstruction?

[JBJS 2017;99:438-45] [Arthroscopy. 2015 Jun;31(6):1185-96]

A

No significant difference

  • DB ACL-R provides significantly better knee stability (by KT arthrometry and pivot-shift testing) than SB ACL-R but no advantages in clinical outcomes or risk of graft failure
103
Q

What is the most common cause of primary ACL reconstruction failures?

[Sport Health 2014; 6(6):504]

A

Technical error

  • Improper tunnel placement outside the native femoral and tibial ACL footprints
104
Q

What is the consequence of improper ACL tunnel placement on the femoral and tibial side?

A
  1. Errors in femoral tunnel placement
  • Too anterior
    • Excessive graft tensioning with flexion leading to loss of knee flexion or stretching of the graft
  • Too posterior
    • Excessive graft tension in extension with laxity in flexion
  • Too vertical
    • Inadequate rotational stability
      1. Errors in tibial tunnel placement
  • Too anterior
    • Notch impingment and loss of extension
  • Too posterior
    • PCL impingement and loss of flexion
  • Too medial/lateral
    • Notch impingement and iatrogenic tibial plateau cartilage damage
105
Q

What evidence is there to support functional bracing post-ACL reconstruction?

[Knee Surg Sports Traumatol Arthrosc (2014) 22:1131–1141]

A
  1. No brace has been validated in the literature to improve patient outcomes or reduce the risk of reinjury
  2. Functional bracing improves proprioception, feelings of stability and confidence
  3. Functional bracing reduces anterior tibial translation (up to 140N) and rotational torques (up to 8Nm)
106
Q

What are potential risk factors for the development of posttraumatic OA following ACL injury?

[Clin Sports Med 32 (2013) 1–12]

A
  1. Meniscus injury
    * Status of the meniscus may be the most important factor for the development of OA regardless of nonoperative or ACL reconstruction
  2. BMI
    * Correlation between BMI and development of OA
  3. Chondral injury
    * May occur at time of injury or secondary to instability
  4. Graft choice
    * BTB graft may have an increased prevalence of OA compared to hamstring graft
  5. Timing of ACL reconstruction
    * Acute surgery may prevent secondary meniscal and chondral injuries
107
Q

At a minimum 10 year followup, what is the risk of developing posttraumatic OA after ACL injury treated nonoperative compared to ACL reconstruction?

[AJSM 2014; 42(9):2242]

A
  1. The relative risk of developing OA after an ACL injury (regardless of associated injuries or treatment) is 3.89 compared to the contralateral uninjured knee
    * i.e. The ACL injured knee is almost 4x more likely to develop OA compared to the contralateral knee
  2. The relative risk of developing OA after ACL reconstruction (regardless of meniscal injury) is 3.62 compared to the contralateral knee
  3. The relative risk of developing OA treated nonoperative (regardless of meniscal injury) is 4.98 compared to the contralateral knee
  4. Therefore, ACL reconstruction reduces the risk of developing OA compared to nonoperative treatment
108
Q

What is the etiology of tunnel widening following ACL reconstruction?

[JAAOS 2010;18:695-706]

A
  1. Mechanical factors
  • Mechanical theory suggests that graft motion within the tunnel can abrade the tunnel edge producing widening
  • Graft position
    • Malpositioned tunnels can lead to increased graft forces being transmitted to the graft-bone interface
    • Eg. anterior tibial tunnel leads to notch impingement and increased force on the graft
  • Fixation method
    • Increased distance between fixation points leads to decreased stiffness and increased graft motion
    • Bungee cord effect
      • Longitudinal graft motion with elongation and retraction of the graft in the tunnel
      • Seen with suspensory fixation compared to aperture fixation
    • Windshield wiper effect
      • Transverse graft motion within the tunnel
      • Seen with fixation not at the aperture of the tunnel
        2. Biological factors
  • Graft sterilization
    • Ethylene oxide sterilization associated with tunnel widening
  • Inflammatroy cytokines
    • May increase osteoclastic activity
  • Implant material
    • Bioabsorbable screws may have greater tunnel widening compared to metal screws
  • Graft type
    • Hamstring autograft shows greater tunnel widening compared to BTB
  • Graft donor
    • May be more widening with allograft compared to autograft
  • Synovial fluid propagation
    • Influx of synovial fluid may delay healing
109
Q

What is acceptable tunnel widening in the first 6-12 months post ACL reconstuction?

[Skeletal Radiol (2013) 42:1489–1500]

A

2mm

110
Q

What radiographic features suggest tunnel widening post ACL reconstruction?

[Skeletal Radiol (2013) 42:1489–1500]

A
  1. Widening >2mm
  2. Loss of parallel walls (cone-shaped)
111
Q

What is the best imaging modality for assessing tunnel widening post ACL reconstruction?

[Skeletal Radiol (2013) 42:1489–1500]

A

CT

112
Q

What are the clinical implications of tunnel widening?

[JAAOS 2010;18:695-706]

A
  1. No implication to clinical outcome
  2. Significant if ACL revision surgery undertaken
113
Q

What are the suggested options for graft choice and implant during revision ACL recon in setting of tunnel widening?

[JAAOS 2010;18:695-706]

A
  1. Allograft achilles tendon
    * If autograft was used for primary
  2. Autograft BTB
    * If allograft was used for primary
  3. Implant – aperature fixation with metallic screw
114
Q

What are the options to manage tunnel widening during ACL revision surgery?

[JAAOS 2010;18:695-706]

A
  1. Single stage (<100% widening)
  • Option in good bone stock
    • Divergent tunnel (funnel) technique
  • Options in poor bone stock
    • Stacked screw
    • Matchstick (bullet) graft
    • Large bone plug graft
      1. Two-stage (>100% widening or tunnel diameter >15mm) [Skeletal Radiol (2017) 46:161–169]
  • Primary bone graft with delayed reconstruction
    • 3-4 months post-bone grafting
115
Q

What is the most common organism causing septic arthritis post-ACL reconstruction?

[J Am Acad Orthop Surg 2013;21:647-656]

A

Staph aureus (31%)

116
Q

In cases of ACL graft contamination (‘dropped graft’) how should you proceed?

[J Am Acad Orthop Surg 2013;21:647-656]

A

Cleanse with NS, followed by chlorhexidine then NS

117
Q

What is the recommended management of septic arthritis post-ACL reconstruction?

[J Am Acad Orthop Surg 2013;21:647-656]

A
  1. Perform arthrocentesis and blood culture prior to antibiotics
    * Septic arthritis suggested if cell count >100,000 (may be as low as 25,000) and cell differential >90 PMNs
  2. Start broad-spectrum antibiotics
    * Can delay until after intra-operative cultures are sent if patient is stable
  3. Immediate arthroscopic I&D
    * Assess graft integrity and viability (probe graft, pivot-shift)
    * Send intraoperative synovial fluid and synovium samples for culture
    * Extensive lavage and synovectomy
    * Graft removal is recommended in the setting of:
    • Significant intrasubstance degeneration
    • Gross evidence of infection compromising the graft
    • Nonfunctional graft as determined by inadequate graft tension or significant pivot shift performed under anesthesia
      * A closed-suction drainage system is typically used postoperatively to assist in wound drainage and is kept in place until there is minimal output
      * Immobilize knee in extension for 24 hours
  4. Second arthroscopic I&D is recommended
    * Repeat synovial fluid aspiration and synovium cultures
  5. 6 weeks of IV antibiotics tailored to culture and sensitivity
  6. When the graft and associated hardware must be removed, a minimum 6-month waiting period after infection eradication is generally recommended before proceeding with graft reimplantation
    * Inection eradication indicated by normalized inflammatory markers and/or normal joint aspirate cell count and culture
118
Q

What is the anatomy of the native Anterolateral Ligament (ALL) of the knee ?

[Arthroscopy 2019 35(2):670-681]

A
  1. Femoral footprint
  • Proximal and posterior to the lateral epicondyle
  • Most variable in the literature but recent trend is proximal and posterior to lateral epicondyle
  1. Tibial footprint
  • Midpoint between the fibular head and Gerdy’s tubercle
  • 4-7mm distal to tibial plateau
  1. Lateral meniscus attachment
    * Between the body and anterior horn of the meniscus
  2. Orientation
    * Fibres run anteroinferior
  3. Length = 30.41 - 59mm
  4. Thickness = 1 - 2mm
119
Q

What structures resist anterolateral rotation (internal tibial rotation) of the knee?

[Clin Sports Med 36 (2017) 135–153]

A
  1. ACL
  2. ALL
  3. Posterior horn of the lateral meniscus
  4. ITB
120
Q

What is the function of a lateral extra-articular tenodesis (LET) combined with an ACL reconstruction?

[Clin Sports Med 36 (2017) 135–153]

A
  1. Protects the ACL graft
    * Demonstrates load-sharing during anterior translation and internal rotation of the tibia
  2. Reduces tibial internal rotation
121
Q

What is a contraindication to LET (in addition to ACL recon)?

[Clin Sports Med 36 (2017) 135–153]

A

Posterolateral corner injury

  • In such cases a tenodesis may tether the tibia in a posterolateral subluxed position
122
Q

What are the relative indications for a LET (in adition to ACL reconstruction)?

[Clin Sports Med 36 (2017) 135–153]

A
  1. Revision ACL reconstruction
  2. Grade 2 or 3 pivot shift (high-grade rotational laxity)
  3. Young age of <25 years
  4. Generalized ligamentous laxity
  5. Genu recurvatum >10o
  6. Returning to pivoting sport (ie, soccer, basketball)
123
Q

Describe the LET procedure – Fowler Kennedy technique

[Clin Sports Med 36 (2017) 135–153]

A
  1. A 5-cm curvilinear incision is placed just posterior to the lateral femoral epicondyle
  2. An 8-cm-long x 1-cm-wide strip of ITB is harvested from the posterior half of the ITB
  3. It is left attached distally at the Gerdy tubercle, freed of any deep attachments to vastus lateralis, released proximally, and a #1 Vicryl whip stitch is placed in the free end of the graft
  4. The LCL is identified and small capsular incisions are made anterior and posterior to the proximal portion and Metzenbaum scissors are placed deep to the FCL to bluntly dissect out a tract for graft passage
  5. The ITB graft is then passed beneath the LCL from distal to proximal
  6. The lateral femoral supracondylar area is then cleared of soft tissue posterior and proximal to the lateral epicondyle
  7. The graft is then held taught but not overtensioned, with the knee at 60 degrees flexion and the foot in neutral rotation to avoid lateral compartment overconstraint
  8. The graft is secured using a small Richards staple and then folded back distally and sutured to itself using the #1 Vicryl whip stitch.
  9. The posterior aspect of the ITB, where the graft was harvested, to avoid overtightening the lateral patellofemoral joint
124
Q

What is the function of the PCL (knee)?

[Arch Bone Jt Surg. 2018; 6(1): 8-18.]

A

Primary restraint to posterior tibial translation at all flexion angles

125
Q

What are the bundles of the PCL?

[JAAOS 2016;24:277-289]

A
  1. Posteromedial
  2. Anterolateral – larger and stronger
126
Q

What structures are part of the PCL complex and may act as secondary restraints to posterior tibial translation?

[JAAOS 2016;24:277-289]

A
  1. Anterior meniscofemoral ligament (of Humphry)
    * From the posterior horn of the lateral meniscus and inserts on the femur anterior to the PM bundle
  2. Posterior meniscofemoral ligament (of Wrisberg)
    * From the posterior horn of the lateral meniscus and inserts on the femur posterior to the PM bundle
127
Q

What is the mechanism of PCL injuries?

[JAAOS 2016;24:277-289]

A

Posteriorly directed force on the proximal tibia with the knee flexed

  • “When the foot is dorsiflexed, force is transmitted through the patella and extensor mechanism. When the foot is plantarflexed, a posteriorly directed force is imparted to the proximal tibia.”
128
Q

What is the normal anterior tibial step-off and what is its significance?

[JAAOS 2016;24:277-289]

A
  1. The medial tibial plateau normally lies approximately 1cm anterior to the medial femoral condyle
  2. This position must be restored when performing a Lachman’s test to prevent false positive and when performing a posterior drawer to prevent a false negative
129
Q

What special tests should be performed when evaluating for PCL injuries?

[JAAOS 2016;24:277-289]

A
  1. Posterior drawer (most sensitive and specific)
    * Graded based on posterior translation of the tibial plateau relative to its position on the contralateral side or relative to the medial femoral condyle (MFC)
  • Grade I
    • 0-5mm posterior translation OR anterior relative to MFC
  • Grade II
    • 6-10mm posterior translation OR even relative to MFC
  • Grade III
    • >10mm posterior translation OR posterior relative to MFC
  1. Quadriceps active test
  • Patient lies supine with the hip flexed to 45° and the knee flexed to 90° and is instructed to slide his or her foot down the table
  • Positive = tibia shifts anterior
  1. Posterior sag
  • Examiner holds the patient’s lower extremity with the hip and knee each flexed to 90°
  • Positive = posterior sagging of the tibia or loss of prominence of the tibial tubercle.
  1. Dynamic posterior shift test
  • Leg is extended from start position of hip and knee flexed to 90°
  • Positive = palpable clunk from reduction of the tibiofemoral joint
130
Q

What associated injury needs to be ruled out in the evaluation of PCL injuries?

[JAAOS 2016;24:277-289]

A

Posterolateral corner injury

  • Dial test
    • With the patient prone an external rotation force is applied to the feet with the knees at 30° and 90°
    • Positive = >10° difference side to side
      • Only at 30° = PLC
      • Both at 30° and 90° = PLC and PCL
131
Q

What additional radiographs can be obtained to evaluate for PCL injuries?

[JAAOS 2016;24:277-289]

A

Kneeling stress views

  • Measurement between a line parallel to the posterior cortex of the tibia and the posterior aspect of Blumensaat’s line is compared between knees
  • 0–7 mm of side-to-side difference in posterior displacement constitutes a partial PCL tear
  • 8–11 mm constitutes an isolated complete PCL tear
  • ≥12 mm of posterior translation constitutes a combined PCL and PLC or PMC knee injury
132
Q

What is the pattern of degenerative changes after a PCL injury?

[JAAOS 2016;24:277-289]

A

Patellofemoral and medial compartment OA

  • Medial compartment experiences more anterior and medial contact pressures
133
Q

What is the management of PCL injuries?

[JAAOS 2016;24:277-289]

A
  1. Nonoperative
  • Isolated grade I and II
  • Grade III with mild symptoms or low activity demands
  1. Operative
    * Acute or chronic grade III who fail nonoperative management
134
Q

What are the two main operative techniques for treating ACL injuries?

[JAAOS 2016;24:277-289] [Arch Bone Jt Surg. 2018; 6(1): 8-18.]

A
  1. Transtibial tunnel
  2. Tibial inlay
    * Involves creating a trough at the tibial attachment of PCL to match with the graft bone plug fixed with a cannulated screw
    * Traditional inlay technique requires an open posteromedial approach (Burks) between the semimembranosus tendon and the medial head of the gastrocnemius muscle
    * Potential benefits
    • Bony healing
    • Avoidance of graft abrasion at the so-called killer tibial turn
    • Large graft sizes
135
Q

Describe the Tibial inlay technique for PCL reconstruction?

[Curr Rev Musculoskelet Med. 2018 Jun; 11(2): 316–319]

A
  1. Patient is prepped and draped in the lateral decubitus position with affected leg up
  2. Outside-in femoral tunnel
  • Patient is positioned supine
  • Diagnostic arthroscopy performed and remnant PCL debrided and footprint identified
  • Medial skin incision is made medial to the patella and through the capsule to identify the guide placement
  • An outside-in arthroscopic guide is used to place the guide pin which is over-reamed (usually 10mm)
  • An 18-gauge metal wire loop is then placed through the femoral tunnel into the posterior aspect of the femoral notch for later graft passage
    3. PCL graft preparation
  • Achilles tendon allograft
    • Bone plug is shaped and predrilled and tapped for a 6.5mm cancellous screw
    • Tendinous portion is tubularized and sized (10mm)
  1. Open tibial inlay
  • Patient is positioned prone
  • Burks posteromedial approach is made between the medial head of gastrocs and semimembranosus
    • Vertical capsulotomy is made and the native PCL insertion is resected with osteotomes and shaped to accept the graft
  • The graft is fixed bone-to-bone with the 6.5mm screw
  • The tendinous portion is shuttled to the femoral tunnel with the 18-gauge wire
  • Wound is closed
    5. Graft tensioning
  • Patient is positioned supine
  • Graft is tensioned with the knee at 90 and interference screw is used for fixation
136
Q

What is the anatomy of the superficial MCL of the knee?

[J Bone Joint Surg Am. 2010;92:1266-80]

A

One femoral and two tibial attachments

  • Femoral attachment = 3.2mm proximal and 4.8mm posterior to the medial epicondyle
  • Proximal tibial attachment = 12.2mm distal to the tibial joint line
    • Primarily to soft tissue over the termination of the anterior arm of the semimembranosus tendon
  • Distal tibial attachment = 61.2mm distal to the tibial joint line
137
Q

What is the function of the superficial MCL?

[Instr Course Lect 2015;64:531–542]

A
  1. Primary restraint against valgus stress at 30o flexion
    * Secondary restraint to external rotation of the tibia
  2. Secondary restraing to tibial internal rotation (along with the posterior oblique ligament) at all flexion angles
138
Q

What is the anatomy of the deep MCL?

[J Bone Joint Surg Am. 2010;92:1266-80]

A

Thickening of the capsule with attachments to the medial meniscus

  • Divided into meniscotibial and meniscofemoral components
  • Meniscofemoral attachment = 12.6 mm distal and deep to the femoral attachment of the superficial MCL
  • Meniscotibial attachment = 3.2 mm distal to the medial joint line
    • Just distal to the tibial articular cartilage
139
Q

What is the anatomy of the posterior oblique ligament (POL)?

[J Bone Joint Surg Am. 2010;92:1266-80]

A
  1. Tibial origin
    * A reflection from the distal semimembranosus tendon insertion on the posteromedial tibia
  2. Femoral origin
    * Central arm of the posterior oblique ligament attaches on the femur 7.7 mm distal and 2.9 mm anterior to the gastrocnemius tubercle
140
Q

How are injuries to the medial knee ligaments classified?

[J Bone Joint Surg Am. 2010;92:1266-80]

A
  1. American Medical Association Standard Nomenclature of Athletic Injuries
    * Severity system
    • Grade I = Localized tenderness and no laxity
    • Grade II = Localized tenderness and partially torn medial collateral and posterior oblique fibers
      • Fibers are still opposed, and there may or may not be pathologic laxity
    • Grade III = Complete disruption and laxity with an applied valgus stress.
      * Laxity system (subjective gapping of the medial joint line)
    • Grade 1+ = 3 to 5 mm
    • Grade 2+ = 6 to 10 mm
    • Grade 3+ >10 mm
  2. Fetto and Marshall Classification [Iowa Orthop J. 2006; 26: 77–90.]
  • Grade I = no valgus laxity at both 0 and 30 degrees of flexion
  • Grade II = valgus laxity at 30 degrees of flexion but stable at 0 degrees of flexion
  • Grade III = valgus laxity at both 0 and 30 degrees of flexion
141
Q

What structures are thought to be intact and disrupted in the Fetto and Marshall Classification?

[J Bone Joint Surg Am. 2010;92:1266-80][Journal of Orthopaedics 14 (2017) 550–554]

A

Grade I

  • No valgus laxity at both 0 and 30o of flexion
  • Both superficial MCL and POL intact

Grade II

  • Valgus laxity at 30o of flexion but stable at 0o of flexion
  • Superficial MCL disrupted and POL intact

Grade III

  • Valgus laxity at both 0 and 30o of flexion
  • Both superficial MCL and POL disrupted
  • Often associated with an ACL injury
142
Q

Where anatomically is the superficial MCL typically injured?

[Sports Med Arthrosc Rev 2015;23:e1–e6)]

A

Femoral insertion > tibial insertion > midsubstance

143
Q

What are the radiographic features of a MCL injury?

A
  1. Stress radiographs [Am J Sports Med. 2010;38:330-8]
  • Isolated superficial MCL
    • Medial joint gapping of 1.7mm at 0° and 3.2mm at 20° of flexion
  • Superficial MCL, deep MCL and POL
    • Medial joint gapping 6.5mm at 0° and 9.8 mm at 20° of flexion
      1. Pellegrini-Stieda syndrome
  • Intraligamentous calcification in the region of the femoral attachment of the MCL caused by the chronic tear of the ligament
144
Q

What are the indications for an MRI with a clinically suspected MCL injury?

[Sports Med Arthrosc Rev 2015;23:e1–e6)]

A
  1. Grade 1 or 2 injuries with concern for associated injuries
  • Relative indications:
    • Effusion
    • Suspected cruciate injury
    • Patellar instability
    • Lateral joint line pain
  1. Grade 3 injuries
  • Assess for concomitant injuries
    • Eg. cruciate injury, stener-like lesion of the knee, etc
145
Q

What are the MRI features of a MCL injury?

[Sports Med Arthrosc Rev 2015;23:e1–e6)]

A

Grade I

  • Ligament swelling and edema
  • Partial tearing with all structures overall intact

Grade II

  • Tearing of some medial structures with others intact
    • Eg. superficial MCL is torn with deep MCL intact, or vice versa

Grade III

  • Complete disruption of the superficial and deep layers
  • Often with significant soft tissue fluid and edema from intra-articular fluid leak

Bone bruises [J Bone Joint Surg Am. 2010;92:1266-80]

  • Lateral tibial plateau and lateral femoral condyle

***Best assessed on the coronal images

146
Q

What is a stener-like lesion of the knee?

[AJR Am J Roentgenol. 2019 Aug 28:1-5.][Clin Orthop Relat Res (2010) 468:289–293]

A

Defined as a distal tear of the superficial MCL with proximal retraction and interposition of osseous or soft-tissue structures between the ligament and its tibial attachment

  • Prevents anatomic healing and with potential for secondary dynamic valgus instability
  • Most commonly stener-like lesions represent interposition of the pes anserinus (83%)
    • May also involve entrapment of the distal tibial portion of the superficial MCL in the tibiofemoral joint or in a reverse Segond fracture (17%)
147
Q

What is the significance of stener-like lesion of the knee?

[AJR Am J Roentgenol. 2019 Aug 28:1-5.]

A
  1. High association with multi-ligament knee injuries
    * Most common associated ligament injury being ACL
  2. Prevents anatomic reduction and healing resulting in valgus instability
148
Q

What is the management of a stener-like lesion of the knee?

[AJR Am J Roentgenol. 2019 Aug 28:1-5.]

A

Acute open reduction and repair

149
Q

What are the indications for nonoperative management of acute medial sided knee injuries?

[Sports Med Arthrosc Rev 2015;23:71–76]

A

Isolated Grade I, II and III MCL injuries

150
Q

What are the indications for operative management of acute medial sided knee injuries?

[Sports Med Arthrosc Rev 2015;23:71–76] [Iowa Orthop J. 2006; 26: 77–90.]

A
  1. Acute repair
  • Large bony avulsion
  • Stener-like lesion of the knee
    • Pes anserine entrapment
    • Intra-articular entrapment
  • MRI finding of complete tibial sided avulsion in athletes
  • Presence of AMRI
  • Presence of valgus instability in 0 degrees of flexion in an underlying valgus knee alignment
  • Associated tibial plateau fracture
    2. Delayed repair
  • Combined ACL or other ligament reconstruction if the EUA shows valgus laxity in 0 degrees of flexion
    3. Augmentation
  • Combined with any repair if local tissue is deficient
    4. Reconstruction
  • Symptomatic chronic valgus laxity/medial instability
    5. Distal femoral varus osteotomy
  • Chronic valgus laxity and valgus bony alignment
151
Q

What are options for acute repair of the superficial MCL and POL?

[Instr Course Lect 2015;64:531–542][Sports Med Arthrosc Rev 2015;23:71–76]

A
  1. Sutures alone
  2. Suture plus suture anchor
  3. Staple
  4. Screw and washer
152
Q

What technique can be used to augment the superficial MCL repair?

[Instr Course Lect 2015;64:531–542]

A

Semitendinosus autograft

  • Released proximally at the musculotendinous junction leaving the distal insertion intact
  • Distal tibial fixation 6 cm from the proximal joint line with two double-loaded suture anchors
  • Graft is then passed deep to the sartorius fascia
  • Femoral fixation in a closed socket tunnel 3.2 mm proximal and 4.8 mm posterior to the medial epicondyle and fixed with an interference screw
  • Proximal tibial attachment is then secured 12 mm distal from the proximal joint line using a double loaded suture anchor
153
Q

What are the key features of a LaPrade superficial MCL and POL reconstruction?

[J Bone Joint Surg Am. 2010;92:1266-80] [Instr Course Lect 2015;64:531–542]

A
  1. Single anteromedial incision
  2. Two separate grafts
  3. 4 reconstruction tunnels at anatomic femoral and tibial insertions of the superficial MCL and POL
  4. Proximal tibial insertion of the superficial MCL is recreated with suture anchors
  5. Superficial MCL is tensioned at 30o flexion
  6. POL is tensioned at 0o flexion
154
Q

What is the anatomy of the lateral collateral ligament of the knee?

[Instr Course Lect 2015;64:531–542]

A
  1. Femoral origin
    * 1.4 mm proximal and 3.1 mm posterior to the lateral epicondyle
  2. Fibula origin
    * 28.4 mm distal to the tip of the fibular styloid
155
Q

What is the function of the LCL (knee)?

[Instr Course Lect 2015;64:531–542]

A
  1. Primary varus stabilizer at 0° and 30° of flexion
  2. Secondary restraint against tibial internal and external rotation
156
Q

What percentage of sporting-related LCL injuries are isolated (no associated pathology)?

[JAAOS 2018 26(6):e120-e127]

A

<2%

157
Q

What physical examination special test is most relevant for assessing the LCL?

[JAAOS 2017;25:280-287]

A

Varus stress test

  • Isolated LCL injury
    • Laxity at 30° and stable at 0° of flexion
  • LCL injury combined with PLC +/- cruciates
    • Laxity at both 30° and 0° of flexion
158
Q

What radiographic findings may be associated with LCL injuries?

[Sports Med Arthrosc Rev 2015;23:10–16] [Instr Course Lect 2015;64:531–542]

A
  1. Arcuate fracture
    * Avulsion of the fibular head or portion of
  2. Varus stress radiograph
    * Isolated LCL injury = 2.7mm lateral gapping
    * LCL and grade III PLC injury = 4.0mm lateral gapping
159
Q

What is the MRI grading system for LCL injuries?

[JAAOS 2018 26(6):e120-e127]

A

Grade I

  • Subcutaneous fluid surrounding the midsubstance of the ligament at one or both insertions

Grade II

  • Partial tearing of ligament fibers at either the midsubstance or one of the insertions, with increased edema in the area

Grade III

  • Complete tearing of ligament fibers at either the midsubstance or one of the insertions
  • Associated with increased edema
160
Q

What are the indications for nonoperative management of LCL injuries?

[JAAOS 2018;26:e120-e127]

A

Grade I and II injury

161
Q

What are the indications for repair of a LCL injury?

[Instr Course Lect 2015;64:531–542]

A

Acute avulsion injury (within 3 weeks)

162
Q

What are the indications for reconstruction of LCL injury?

[Instr Course Lect 2015;64:531–542]

A
  1. Acute Grade III midsubstance tears or attenuation
  2. Chronic LCL injury with symptomatic instability
163
Q

What are the indications for valgus producing HTO in context of LCL tear?

[Instr Course Lect 2015;64:531–542]

A

Chronic symptomatic LCL injury with varus deformity

  • HTO should precede reconstruction
    • May eliminate need for reconstruction in some cases
164
Q

What are the 3 key anatomical structures that make up the posterolateral corner (PLC) of the knee?

[Rev Bras Ortop. 2015 50(4): 363–370]

A
  1. Lateral collateral ligament
  2. Popliteus tendon
  3. Popliteofibular ligament
165
Q

Other than the primary components of the PLC, what other structures stabilize the posterolateral corner of the knee?

[JAAOS 2008;16:506-518] [Sports Med Arthrosc Rev 2015;23:2–9)]

A
  1. Posterolateral capsule
  2. Lateral meniscus
  3. Iliotibial band
  4. Fabellofibular ligament
  5. Long and short heads of the biceps femoris
  6. Lateral head of gastrocnemius muscle
166
Q

Describe the anatomy of the LCL, popliteus tendon and the popliteofibular ligament?

[Sports Med Arthrosc Rev 2015;23:2–9)]

A
  1. Lateral collateral ligament
  • Proximal attachment
    • Proximal and posterior to the lateral epicondyle of the femur
  • Distal attachment
    • Lateral aspect of the fibular head
      • 8.2mm posterior to the anterior margin of the fibular head and 28.4mm distal to the fibular styloid tip
        2. Popliteus tendon
  • Proximal attachment
    • Anterior and distal to the lateral epicondyle
      • At the anterior 1/5 of the popliteal sulcus, just posterior to the lateral femoral condyle articular cartilage
      • Attachment is 18.5mm oblique from the LCL femoral attachment
  • Distal attachment
    • Proximal posteromedial aspect of the tibia
  • ***Note: The tendon passes from intra-articular through the popliteal hiatus to become extra-articular
    • Gives off 3 popliteomeniscal fascicles to anchor to the lateral meniscus
      1. Popliteofibular ligament
  • Proximal attachment
    • Musculotendinous junction of the popliteus
  • Distal attachment
    • Posteromedial downslope of the fibular styloid process
167
Q

What is the consequence of a missed PLC injury?

[Rev Bras Ortop. 2015 50(4): 363–370]

A
  1. Failure of cruciate reconstructions
  2. Recurrent instability
168
Q

What are the special tests for the PLC?

[JAAOS 2017;25:280-287]

A
  1. External rotation recurvatum test
  • Lower leg is picked up by the great toe
  • Relative hyperextension, tibial external rotation and knee varus compared to the contralateral side
  1. Posterolateral rotary drawer test
  • With the knee flexed to 90°, the hip flexed to 45°, and the foot fixed in slight external rotation (usually best at 15°), a posteriorly directed force is applied through the tibial tuberosity
    • The PCL is relaxed in this position, allowing rotary and translator laxity
  • With an isolated PLC injury
    • More rotatory instability with slight ER than with neutral rotation because the PCL provides more translational stability with neutral rotation
  • With an isolated PCL injury
    • More translatory instability than rotary instability will be present
  1. Dial test
  • Patient prone, external rotation applied to foot and thigh foot angle compared side to side at 30 and 90 of knee flexion (Positive = 10o difference)
    • Increased at 30° = isolated PLC
    • Increased at 30° and 90° = PLC and PCL
      1. Standing apprehension test
  • Patient stands with the knee slightly bent and internally rotates the torso away from the leg, producing an internal rotation of the femur on the tibia
  • Positive = Apprehension or instability
169
Q

What classifications have been proposed to grade PLC injuries?

[JAAOS 2008;16:506-518]

A
  1. Hughston classification
  • Severity graded on varus laxity
  • Grade I = 0 to 5mm
    • Sprains without tensile failure of any capsule-ligamentous structures
  • Grade II = 6-10mm
    • Partial injuries with minimal abnormal laxity
  • Grade III = >10mm
    • Complete disruptions with significant laxity
      1. Fanelli classification
  • Severity graded on external rotation and varus laxity
  • Type A
    • Isolated rotational injury to the PFL and popliteus tendon complex
  • Type B
    • Rotational injury with a mild varus component
    • Represents an injury to the PFL and popliteus tendon complex, as well as attenuation of the LCL
  • Type C
    • Posterolateral instability with significant rotational and varus component secondary to complete disruption of the PFL, popliteus tendon complex, LCL, lateral capsule, and cruciate ligament or ligaments
      1. Modified Hughston classification
  • Severity graded on ER and varus laxity
  • Grade I
    • Minimal instability
    • Either varus or rotational instability of 0 to 5mm or 0° to 5°
  • Grade II
    • Moderate instability (6 to 10mm or 6° to 10°)
  • Grade III
    • Significant instability (>10 mm or >10°)
170
Q

What are the indications for nonoperative management of PLC injuries?

[Rev Bras Ortop. 2015 50(4): 363–370]

A

Acute, isolated Grade I and II PLC injuries

171
Q

What are the indications for operative management of PLC injuries?

[Rev Bras Ortop. 2015 50(4): 363–370]

A
  1. Isolated grade III PLC injuries
  2. Combined PLC injuries
  3. Chronic symptomatic PLC injury failing nonoperative management
172
Q

What are the indications for repair (vs. reconstruction) of PLC injuries?

[Rev Bras Ortop. 2015 50(4): 363–370]

A

Acute (<3 weeks) LCL and popliteus tendon avulsions, without midsubstance injury

173
Q

In general, in a PLC injury what type of reconstruction should be performed and of what structures?

[Knee Surgery, Sports Traumatology, Arthroscopy (2019) 27:2520–2529]

A

Anatomical reconstruction

  • Reconstruction of primary PLC components
    • LCL, popliteus tendon and PFL
  • Repair of secondary structures (hybrid construct)

***Note: Valgus producing HTO should precede or occur concurrently with reconstruction in varus knees

174
Q

What are the anterior, middle and posterior thirds of the medial side of the knee as described by Robinson?

[JAAOS 2017;25:752-761]

A

Anterior 1/3

  • Medial border of the patellar tendon to the anterior border of the superficial MCL

Middle 1/3

  • Width of the superficial MCL

Posterior 1/3

  • Posterior border of the superficial MCL to the medial border of the medial head of gastrocnemius
  • Posterior 1/3 = posteromedial corner
175
Q

What are the 5 components of the PMC?

[JAAOS 2017;25:752-761]

A
  1. Posterior oblique ligament
  2. Semimembranosus tendon and its expansions
  3. Oblique popliteal ligament
  4. Posteromedial joint capsule
  5. Posterior horn of the medial meniscus
176
Q

Describe the anatomy of the posterior oblique ligament (POL)?

[JAAOS 2017;25:752-761]

A
  1. Proximal attachment
    * Distal and posterior to the adductor tubercle
  2. Runs distally and posterior at 25° to the superficial MCL fibres
  3. Distally has 3 arms
  • Superficial
  • Central (tibial)
    • Largest and thickest
    • Reinforces the posteromedial capsule
    • Attaches to the posteromedial aspect of the medial meniscus and adjacent tibia
    • Main structure requiring repair or reconstruction
  • Capsular
177
Q

What are the 5 major arms or expansions of the semimembranosus tendon?

[JAAOS 2017;25:752-761]

A
  1. Pars reflecta (anterior arm)
    * Inserts into medial tibia
  2. Direct posteromedial tibial insertion (primary attachment)
    * Inserts into posteromedial tibia (tuberculum tendinis)
  3. OPL insertion
  4. POL insertion
  5. Popliteus aponeurosis expansion
178
Q

Describe the anatomy of the oblique popliteal ligament?

[JAAOS 2017;25:752-761]

A

Extends from the main semimembranosus tendon passing laterally and proximal to attach to the posterior capsule (meniscofemoral portion), fabella, and plantaris muscle

  • Difficult to distinguish from the posterior capsule
179
Q

What attachments are present on the posterior horn of the medial meniscus?

[JAAOS 2017;25:752-761]

A
  1. Posteromedial capsule (deep MCL)
  2. POL
  3. Semimembranosus expansion
180
Q

What is the most frequently injured structure in PMC injuries?

[JAAOS 2017;25:752-761]

A

POL

181
Q

What are the 3 major patterns of PMC injury?

[JAAOS 2017;25:752-761]

A
  1. POL + semimembranosus tendon [capsular arm] (70%)
  2. POL + complete peripheral meniscus detachment (30%)
  3. POL + semimembranosus tendon + peripheral meniscus detachment (19%)

NOTE: isolated PMC injury is rare

182
Q

What are the findings on physical examination associated with PMC injury?

[JAAOS 2017;25:752-761]

A
  1. AMRI (anteromedial rotatory instability)
  • Hallmark of PMC injury
  • Characterized by anterior subluxation of the anteromedial tibia on the femoral condyle
  • Valgus stress at 30° flexion with foot externally rotated
    • Positive = medial joint gapping and anterior subluxation of the medial tibial plateau relative to the femur
      • Correlates with combined MCL and PMC
        2. Anterior drawer with foot 15° externally rotated
  • Positive = anteromedial tibial plateau subluxation
    • Indicates PMC injury
      1. Valgus stress at 0°
  • Positive = gapping of medial joint line
    • Suggests MCL, cruciate and PMC injury
  1. Posterior drawer with tibia in neutral and internal rotation
  • Positive = equal posterior translation
    • PCL and PMC injury
  • Decreased posterior translation with internal rotation suggests isolated PCL injury
183
Q

What are the indications for surgical repair or reconstruction of the PMC?

[JAAOS 2017;25:752-761]

A
  1. Multiligamentous injuries with AMRI
  2. Medial gapping or instability with valgus stress at full extension
  3. Positive posterior drawer with internal rotation
184
Q

When can PMC repair be considered?

[JAAOS 2017;25:752-761]

A

Acute injury

  • Particularly with distal tibial sided MCL and PMC tears
185
Q

When should PMC reconstruction be considered?

[JAAOS 2017;25:752-761]

A
  1. PMC tissue is not favourable (attenuated, midsubstance)
  2. Chronic, symptomatic PMC injuries
186
Q

What is the anatomic reconstruction of the PMC?

[JAAOS 2017;25:752-761]

A

2 grafts to reconstruct

  • Superficial MCL
    • Distal tunnel 6cm below joint line
    • Proximal tunnel just proximal and anterior to the medial epicondyle
    • Tensioned in 20° flexion and neutral rotation
    • Proximal tibial attachment is made with a suture anchor 12-13mm distal to joint line
  • POL
    • Distal tunnel just anterior to the direct arm attachment of the semimembranosus
    • Proximal tunnel 8mm distal and 3mm anterior to the gastrocnemius tubercle
    • Tensioned and fixed in full extension and neutral rotation
187
Q

What are the consequences of an unrecognized PMC injury?

[JAAOS 2017;25:752-761]

A
  1. Chronic valgus laxity
  2. Late graft failure after ACL reconstruction
188
Q

What is the blood supply to the menisci?

[Am J Sports Med 1982; 10(2):90]

A
  1. Primary = medial and lateral geniculate arteries (both superior and inferior)
    * Gives rise to a perimeniscal capillary plexus in the synovial and capsular tissue supplying the peripheral meniscus (10-30%)
  2. Secondary - middle geniculate artery
  • Within the synovial sheath of the cruciate ligaments
  • Supplies the anterior and posterior horns for short distance
  1. Relative avascular area of the lateral meniscus at the posterolateral aspect adjacent to the popliteus tendon
  2. Three vascular zones [AAOS comprehensive review 2, 2014]
  • Red-red – peripheral 1/3 (vascular)
  • Red-white – middle 1/3 (avascular)
  • White-white – central 1/3 (avascular)
189
Q

What is the innervation of the menisci?

[Clinical Anatomy 28:269–287 (2015)]

A
  1. Recurrent peroneal branch of the common peroneal nerve
  2. Mechanoreceptors and free nerve endings located in the anterior and posterior horns and peripheral 1/3-2/3
190
Q

What is the function of the meniscus?

[Clinical Anatomy 28:269–287 (2015)]

A
  1. Load transmission
    * Converts compressive forces into concentric forces
    • Hoop stresses transmitted to bony anchors of meniscal horns by circumferential fibres of meniscus
    • Diminishes forces on articular cartilage
      * ​40-60% of the load in extension and up to 90% in flexion
  2. Shock absorption
  3. Stability (limits motion in all directions)
  4. Proprioception
  5. Joint lubrication and nutrition (?theoretical)
191
Q

What are the types of meniscus tears?

[J Am Acad Orthop Surg 2013;21:204-213]

A
  1. Radial
  2. Vertical/longitudinal
  3. Horizontal
  4. Bucket-handle
  5. Oblique/flap
  6. Complex/degenerative
  7. Meniscal root
192
Q

What are the indications for meniscal repair?

[J Am Acad Orthop Surg 2013;21:204-213]

A
  1. Tear >1cm but <4cm in length
    * <1cm = stable
    * >4cm = high failure rate
  2. Red-red zone tears
    * Most important intrinsic factor in healing
    * Tears within 2mm of vascular rim have highest rate of healing
    * Tears >4mm from rim have high failure rates

​3. Vertical/longitudinal tears

  • Most amenable to repair
    4. Age <40 years
  • Success controversial (can be successful in older patients)
  • Effect important in younger patients
    5. Acute tears (<6 weeks)
  • Controversial, conflicting data on failure rates
    6. Concurrent ACL reconstruction
  • Concurrent meniscal repair and ACL reconstruction are equal or more successful than meniscal repair with intact ACL
    • Can also be staged
      1. No mechanical malalignment
      2. Radial tears that extend entire width of meniscus
      3. Meniscal root tears
193
Q

What are the 3 main options for meniscal repair?

[J Am Acad Orthop Surg 2013;21:204-213]

A
  1. Inside-out (gold standard)
  2. Outside-in
  3. All inside
194
Q

For the inside out technique of meniscal repair describe the medial and lateral approach?

[Arthrosc Tech. 2017 Aug; 6(4): e1221–e1227.]

A
  1. Medial approach
  • 4cm vertical incision posterior to MCL
    • 1/3 above joint and 2/3 below joint
  • Sartorial fascia is opened inline
  • Pes anserinus tendons are taken distal
  • Interval between the medial head of gastrocs and capsule is developed staying proximal to semimembranosus tendon
    • Spoon is inserted in this interval
      1. Lateral approach
  • 4cm oblique incision starting at Gerdy’s extending proximal and anterior
  • ITB is split in line with its fibres
  • Staying posterior to LCL and anterior to the biceps femoris the interval between the lateral head of gastrocs and the capsule is developed
    • Spoon is inserted in to this interval
195
Q

What is the function of the meniscus root?

[JBJS REVIEWS 2016;4(8):e2] [AJSM 2014; 42(12): 3016]

A

Dissipate axial loads by conversion to hoop stresses

  • Axial load causes circumferential fibres to stretch and meniscus to extrude
  • Tensile ‘hoop stress’ resists this extrusion in the presence of intact meniscal roots
  • Distribution of hoop stresses by the circumferential fibers helps to transmit relatively even axial loads across the joint surfaces
196
Q

What is the consequence of meniscal root tears?

[JBJS REVIEWS 2016;4(8):e2]

A
  1. Meniscal extrusion
  2. Loss of hoop stress
  3. Increased tibiofemoral contact pressures
    * Equivalent to total meniscectomy
  4. Accelerated degenerative changes
197
Q

What are the typical clinical symptoms following a meniscal root tear?

[JBJS REVIEWS 2016;4(8):e2] [AJSM 2014; 42(12): 3016]

A
  1. Joint line tenderness
  2. Pain in deep knee flexion
  3. Positive McMurray test
  4. Effusion
  5. Locking
  6. Giving way
198
Q

What are the MRI signs indicative of a meniscal root tear?

[JBJS REVIEWS 2016;4(8):e2]

A
  1. Radial tear of the meniscal root on axial imaging
  2. Truncation sign
    * Vertical linear defect in the meniscal root on coronal imaging
  3. Ghost sign
    * Increased signal in the meniscal root on sagittal sequences
  4. Meniscal extrusion >3 mm outside the peripheral margin of the joint on coronal imaging
  5. Ipsilateral tibiofemoral compartment bone marrow edema and insufficiency fractures are commonly noted in the presence of posterior meniscal tears
199
Q

What meniscal root has the highest incidence of tears?

[JBJS REVIEWS 2016;4(8):e2]

A

Posterior root of the medial meniscus

  • Least mobile of all roots
200
Q

What is the classification of meniscal root tears?

[Arch Bone Jt Surg. 2018; 6(4): 250-259.]

A

LaPrade classification

  • Type I
    • Partial root tear
  • Type II
    • Complete radial root tear
  • Type III
    • Complete root tear + bucket handle tear
  • Type IV
    • Oblique tear into root attachment
  • Type V
    • Root avulsion fracture
201
Q

What are the indications for nonoperative and operative management of meniscal root tears?

[JBJS REVIEWS 2016;4(8):e2]

A

Nonoperative

  • Sedentary lifestyle
  • Extensive medical comorbidities
  • Advanced signs of osteoarthritis
    • Outerbridge grade 3 or 4
  • Joint-space narrowing
  • Marked varus malalignment (>5°)
  • Chronic, degenerative, irreparable tears

Operative

  • Healthy individuals
  • Minimal or no degenerative changes
    • Outerbridge grade 1 or 2
  • Minimal to no joint-space narrowing
  • Normal mechanical alignment
  • Intact cruciate ligaments
202
Q

What are the indications for meniscal repair vs. partial meniscectomy in context of a root tear?

[JBJS REVIEWS 2016;4(8):e2]

A
  1. Partial meniscectomy
  • Chronic tears with advanced degenerative changes
    • Outerbridge grade 3 or 4
  1. Meniscal root repair
  • Acute tears
    • Goal of preventing arthritis
  • Chronic tears with no or little articular cartilage wear
    • Outerbridge grade 1 or 2
    • Goal of symptomatic relief
203
Q

What is the goal of operative treatment for a meniscal root tear?

[JBJS REVIEWS 2016;4(8):e2]

A

Restore an anatomical attachment of the meniscal root to bone that is capable of converting axial weight-bearing loads into hoop stresses

  • Thereby improving symptoms and function and ultimately preventing or delaying onset of degenerative changes
204
Q

What are the two main meniscal root repair techniques?

[JBJS REVIEWS 2016;4(8):e2]

A
  1. Transtibial pullout repair
  • Anatomical position of the meniscal root is identified, debrided, and prepared for repair
  • An ACL tibial drill guide is utilized to drill a guide pin or a retro-cutting reamer into position through a small incision at the anteromedial aspect of the proximal aspect of the tibia
  • # 2 nonabsorbable sutures are passed through the substance of the torn meniscal root with a suture-passage device
  • Suture limbs are shuttled through the transtibial tunnel, tensioned, and fixed with use of whatever fixation construct the surgeon prefers with the knee in 30° of flexion
    • Typically a cortical button or screw and washer
      1. Suture anchor repair
  • Utilizes accessory posteromedial and posterolateral portals
  • Under arthroscopic visualization, the anatomical position of the meniscal root is identified, debrided, and prepared for repair
  • A double-loaded suture anchor is inserted at the meniscal root site and suture passage is performed with use of a suture lasso through the accessory portal or with use of a rotator cuff-type suture-passage device through the anteromedial or anterolateral portal
  • Once passed, the suture limbs are tied arthroscopically with the knee in 30° of flexion, with the arthroscopic knots being kept posterior, away from the articular surfaces of the affected compartment
205
Q

What are the complications associated with meniscal root repair techniques?

[AJSM 2014; 42(12): 3016]

A
  1. Transosseous fixation
  • Bone tunnels may interfere with concomitant ligament reconstruction
  • Risk of suture abrasion within bony tunnel
  • Risk of creep in the suture
    • Decreases strength of repair
      1. Suture anchor repair
  • Anchor pullout
  • Technically difficult
  1. Other
  • Retear and progression of arthritis
  • Infection
  • Arthrofibrosis
  • DVT
  • Iatrogenic injury to cruciates
  • Iatrogenic injury to posterior neurovascular structures
206
Q

What are the most common locations for OCD lesions in the knee?

[Sports Med Arthrosc Rev 2016;24:44–49]

A
  1. 70% - posterolateral aspect of the medial femoral condyle
  2. 15% - inferior central aspect of the lateral femoral condyle
  3. 5-10% - inferior medial aspect of the patella
  4. <1% - trochlea
207
Q

What other pathology is associated with an OCD lesion of the lateral femoral condyle?

A

Discoid lateral meniscus

208
Q

What are the two broad categories of OCD?

[Curr Rev Musculoskelet Med (2015) 8:467–475]

A
  1. Juvenile (open physis)
  2. Adult (closed physis)
209
Q

What is Wilson’s sign (Knee OCD)?

[Curr Rev Musculoskelet Med (2015) 8:467–475]

A

The knee is extended from 90-30 while internally rotating the tibia

  • Positive = reproduction of symptoms with internal rotation and relief with external rotation
210
Q

What are MRI features of an unstable OCD lesion?

[Curr Rev Musculoskelet Med (2015) 8:467–475]

A
  1. High-signal intensity behind the lesion
  2. Cystic area beneath the lesion
  3. High-signal intensity through the articular cartilage
  4. Focal articular defect
211
Q

What factors are predictive of poor prognosis in OCD lesions in the knee?

[Sports Med Arthrosc Rev 2016;24:44–49]

A
  1. High signal line between the OCD lesion and underlying bone on T2-MRI (unstable)
  2. Skeletal maturity (older age)
  3. Larger lesion
  4. Patellar and lateral femoral condyle OCD lesions
  5. Cyst ≥1.3mm behind the lesion
212
Q

What is the classification of OCD lesion of the knee?

[Orthobullets]

A

Clanton Classification

  • Type I - depressed osteochondral fracture
  • Type II - fragment attached by osseous bridge
  • Type III - detached non-displaced fragment
  • Type IV - displaced fragment
213
Q

What are the indications for nonoperative management of an OCD lesion in the knee?

[Curr Rev Musculoskelet Med (2015) 8:467–475]

A

Stable lesion

214
Q

What is the protocol for nonoperative management?

[Bone Joint J 2016;98-B:723–9]

A

***No consensus

  1. TTWB 4 weeks
  2. Activity restriction (no running and sport) for 8 weeks
  3. MRI at 12 weeks
    * If no improvement in symptoms = consider surgery
    * If symptoms resolved and MRI improved = progress activity with full return after 4 weeks
    * If symptoms improved but MRI is unchanged = continue activity restriction and repeat MRI in 12 months
    • If no improvement = surgery
215
Q

What are the indications for operative management of an OCD lesion in the knee?

[Curr Rev Musculoskelet Med (2015) 8:467–475]

A
  1. Unstable lesion
  2. Displaced lesion
  3. Failure of nonoperative management
    * 6 months for juvenile, 3-6 months for adult
  4. Symptomatic loose bodies
216
Q

What are the surgical options for OCD of the knee?

[Bone Joint J 2016;98-B:723–9] [Curr Rev Musculoskelet Med (2015) 8:467–475]

A
  1. Stable symptomatic lesion
  • Retrograde or transarticular drilling
    • Equivalent effectiveness
  1. Unstable, nondisplaced
    * Internal fixation with bioabsorbable or metal pins, nails or screws
  2. Unstable, nondisplaced with cyst
    * bone grafting and internal fixation
  3. Unstable, displaced
  • internal fixation of fragment
    • Fragment trimming due to hypertrophy
  1. Unsalvageable fragment
    * Excision = small fragments (<2cm)
    * Microfracture
    * ACI/MACI
    * Osteochondral autograft or allograft
  2. Consider osteotomy or patellar realignment to offload the lesion
217
Q

What are the indications and contraindications for articular cartilage repair and restoration procedures?

[JAAOS 2017;25:321-329]

A

Indications

  • Outerbridge or International Cartilage Repair Society grade III or IV focal chondral or osteochondral defects

Contraindications

  • Inflammatory arthritis
  • Lower grade lesions
  • Inability to comply with postoperative protocol
218
Q

What are the general surgical recommendations for articular cartilage lesions based on size of defect?

[Sports Health 2015; 8(2):153]

A

<2cm2 – microfracture OR OATs

  • OATS better in higher demand patient

2-4cm2 - OATs or ACI

>4cm2 – ACI or osteochondral allograft (better in defects with bone loss or deformity)

219
Q

What is the technique for microfracture of articular cartilage lesions?

[JBJS 2010;92:994-1009] [JAAOS 2017;25:321-329]

A
  1. Create a contained lesion with stable shoulders
  • Remove the calcified cartilage layer at the base of the lesion
  • Prepare the channels by perforating the subchondral bone with a 45° awl approx. 3-4mm in depth
    • Fat droplets from the marrow should be visualized, blood should be visualized with water off
    • Space the channels 3-4mm apart
  • Allows for clot formation and migration of marrow-derived mesenchymal stem cells promoting fibrocartilage repair
  1. Postoperative
  • nonWB for 6 weeks with crutches
  • CPM from 0-60° for 6 weeks, 6-8 hours per day
  • Full ROM and closed chain exercises at 2 months
  • Full return to activity at 3 months
220
Q

What are the advantages and disadvantages of microfracture for articular cartilage lesions?

[JBJS 2010;92:994-1009]

A

Advantages

  • Technically straightforward
  • Low cost
  • Minimal morbidity

Disadvantages

  • Fibrocartilage rather than hyaline cartilage
    • Results deteriorate over time
221
Q

What is the technique for autologous osteochondral transplantation (aka. Mosaicplasty, osteoarticular transfer system – OATS) for articular cartilage lesions?

[JAAOS 2017;25:321-329]

A

Multiple small autogenous osteochondral plugs are harvested from less weightbearing areas and transferred to fill a defect

  • Plugs are approx. 10mm in depth, impacted in donor site to match the height of the adjacent cartilage
  • Periphery of femoral condyles and intercondylar notch are donor sites
222
Q

What are the advantages and disadvantages of autologous osteochondral transplantation (OATS) for articular cartilage lesions?

[JBJS 2010;92:994-1009] [JAAOS 2017;25:321-329]

A

Advantages

  • Hyaline cartilage transferred
  • Less postoperative restrictions
  • Single procedure

Disadvantages

  • Limited availability of graft
  • Donor site morbidity
  • Different thickness and mechanical properties of cartilage from donor to recipient site
  • Graft subsidence
  • Dead space between grafts
  • May require arthrotomy
223
Q

What is the technique for osteochondral allograft transplantation for articular cartilage lesions?

[JBJS 2010;92:994-1009]

A

Cadaver graft consisting of intact, viable articular cartilage and subchondral bone is transferred in the defect

224
Q

What are the advantages and disadvantages of osteochondral allograft transplantation?

[JBJS 2010;92:994-1009]

A

Advantages

  • No donor site morbidity
  • Can fill large defects
  • Hyaline cartilage transferred
  • Single stage
  • Can achieve precise surface architecture
  • Eliminates dead space

Disadvantages

  • Limited graft availability
  • High cost
  • Risk of immunological reaction and disease transmission
  • Possible incomplete graft incorporation
  • Technically demanding machining and sizing the allograft
225
Q

What is the technique for autologous chondrocyte implantation (ACI)?

[JBJS 2010;92:994-1009] [JAAOS 2017;25:321-329]

A

Two stage procedure

  • First for tissue harvest
    • 100-300mg of articular cartilage from nonWB surface
    • Sent for culture and expansion of donor chondrocytes
  • Second for cell implantation into the prepared recipient site
    • 3-8 weeks after the first stage
    • Cells are injected beneath a collagen membrane or periosteal patch sutured over the defect
226
Q

What are the advantages and disadvantages of ACI?

[JBJS 2010;92:994-1009] [Sports Health 2015; 8(2):153]

A

Advantages

  • Hyaline cartilage produced

Disadvantages

  • Two stage procedure
  • Technically demanding
  • High cost
  • Requires arthrotomy
227
Q

What is difference between ACI and matrix-associated chondrocyte implantation (MACI)?

[JBJS 2010;92:994-1009]

A

Chondrocytes are incorporated into a collagen membrane eliminating the need for periosteal harvest and makes for a more even cell distribution on the membrane

228
Q

What are the standard and accessory portals for knee arthroscopy?

A
  1. Standard
  • Anterolateral
  • Anteromedial
  1. Accessory [JBJS 2006; 88(4):110]
  • Posteromedial
    • Location
      • Soft spot between MCL, medial head of gastrocs and semitendinosus tendon
    • Technique
      • Perform with knee at 90
      • Knee joint distended
      • Direct visualization with transcondylar notch view
      • Soft spot is localized with a spinal needle
      • Superficial incision
      • Blunt dissection with hemostat and joint capsule is penetrated
    • Risk
      • Saphenous nerve (sartorial branch)
  • Posterolateral
    • Location
      • Soft spot between LCL, lateral head of gastrocs, posterolateral tibial plateau and biceps femoris
    • Technique
      • Perform with knee at 90
      • Knee joint distended
      • Direct visualization with transcondylar notch view
      • Soft spot is localized with a spinal needle
      • Superficial incision
      • Blunt dissection with hemostat and joint capsule is penetrated
    • Risk
      • Common peroneal nerve
  • Posterior transeptal portal
    • Location
      • Involves resection of the posterior septum from the posterolateral portal to enter the posteromedial compartment under visualization from the posteromedial portal
        • Posterior septum is a triangle-shaped two-layer synovial reflection that divides the posterior aspect of the knee into posteromedial and posterolateral compartments
        • Bounded by the posterior cruciate ligament anteriorly, the posterior portion of the femoral intercondylar notch superiorly, and the posterior aspect of the capsule posteriorly
    • Risk
      • Popliteal artery (inferiorly) and middle geniculate artery (superiorly)
229
Q

What are ways to control intraoperative bleeding during arthroscopy?

[CORR course]

A
  1. Increased pump pressure
  2. Reduce outflow suction
  3. Asd epinephrine to fluid
  4. Controlled hypotension (anaesthesia)
  5. Tourniquet
  6. Drive arthroscope to bleeding source
  7. Cautery
  8. TXA
230
Q

What are the avulsion fractures found at the knee?

A